How do I determine the magnitude of torque from a cross product?

  • Thread starter Thread starter smatt_31
  • Start date Start date
  • Tags Tags
    Magnitude Torque
Click For Summary
To determine the magnitude of torque from the cross product, the calculation involves evaluating the expression k = (-15(-200 sin theta)) - (12(-200 cos theta)), which simplifies to k = 3000(sin theta) + 2400(cos theta). The coefficient of k represents the torque's magnitude, and it is not necessary to take the square root of this value. Instead, the magnitude is simply the absolute value of the coefficient, provided it is positive. The discussion clarifies that theta's undefined nature does not complicate the calculation of torque magnitude. Understanding the relationship between the cross product and torque is essential for solving such problems.
smatt_31
Messages
6
Reaction score
0
[SOLVED] the magnitude of torque

Homework Statement


Determine the magnitude of the torque about A by evaluating lABxFl (the magnitude of vector AB cross vector F) where AB = -15i + 12j and F= -200lbs (cos theta)i - 200lbs (sin theta)j

Homework Equations


cross product and maybe the magnitude of torque

The Attempt at a Solution


so i did the cross product and ended up with
k = (-15(-200 sin theta))-(12(-200cos theta)) whick i simplified
to 3000(sin theta) + 2400(cos theta)

so k = 3000(sin theta) + 2400(cos theta)

so am I done or should I take the sqrt of the values = to k ??
the problem i have with this is theta is undefined and i would end up with a more complicated expression.
so what do i do
 
Physics news on Phys.org
smatt_31 said:

Homework Statement


Determine the magnitude of the torque about A by evaluating lABxFl (the magnitude of vector AB cross vector F) where AB = -15i + 12j and F= -200lbs (cos theta)i - 200lbs (sin theta)j

Homework Equations


cross product and maybe the magnitude of torque

The Attempt at a Solution


so i did the cross product and ended up with
k = (-15(-200 sin theta))-(12(-200cos theta)) whick i simplified
to 3000(sin theta) + 2400(cos theta)

so k = 3000(sin theta) + 2400(cos theta)

so am I done or should I take the sqrt of the values = to k ??
the problem i have with this is theta is undefined and i would end up with a more complicated expression.
so what do i do
That is not "k= ". What you have is the coefficient of k (unit z-vector). The magnitude is the value of that coefficient. If you wanted to be real technical, the "magnitude" is the square root of the square of that: its absolute value and so just what you have as long as it is positive.
 
Question: A clock's minute hand has length 4 and its hour hand has length 3. What is the distance between the tips at the moment when it is increasing most rapidly?(Putnam Exam Question) Answer: Making assumption that both the hands moves at constant angular velocities, the answer is ## \sqrt{7} .## But don't you think this assumption is somewhat doubtful and wrong?

Similar threads

  • · Replies 14 ·
Replies
14
Views
2K
  • · Replies 1 ·
Replies
1
Views
4K
Replies
1
Views
1K
  • · Replies 11 ·
Replies
11
Views
1K
Replies
2
Views
2K
  • · Replies 1 ·
Replies
1
Views
1K
  • · Replies 8 ·
Replies
8
Views
12K
  • · Replies 2 ·
Replies
2
Views
8K
  • · Replies 8 ·
Replies
8
Views
2K
Replies
4
Views
8K